Full Rank Matrix: Determinant Condition | Rank-Nullity Theorem

Click For Summary
A matrix A is of full rank if and only if the determinant condition ad - bc ≠ 0 is satisfied, indicating that A is nonsingular and has an inverse. By the rank-nullity theorem, a full rank matrix has a kernel dimension of 0, confirming that the column vectors are linearly independent. Conversely, if ad - bc = 0, the matrix is singular, and the column vectors must be linearly dependent. The discussion also explores demonstrating the linear dependence of vectors without relying on determinants. The relationship between the determinant and linear independence is crucial for understanding matrix properties.
squenshl
Messages
468
Reaction score
4

Homework Statement


Show that the matrix ##A## is of full rank if and only if ##ad-bc \neq 0## where $$A = \begin{bmatrix}
a & b \\
b & c
\end{bmatrix}$$

Homework Equations

The Attempt at a Solution


Suppose that the matrix ##A## is of full rank. That is, rank ##2##. Then by the rank-nullity theorem, the
dimension of the kernel is ##0##. This implies that there exists an inverse ##A^{-1}## but this will only occur if ##ad-bc \neq 0## otherwise our matrix ##A## will be singular. On the other hand, suppose ##ad-bc \neq 0##. Hence, ##A## is nonsingular and there exists an inverse ##A^{-1}## but this will occur only when the dimension of the kernel is ##0##, that is, of rank ##n = 2##.
 
Physics news on Phys.org
Your matrix has rank 2 iff the column vectors are linearly independent.
Can you show that the determinant of the matrix is zero iff the column vectors are linearly dependent ?
 
Sorry $$A=\begin{bmatrix}
a & b \\
c & d
\end{bmatrix}$$ if that even matters!

How could I do this without using anything on determinants?
 
Show that ad - bc = 0 iff ##\vec u = (a,c) ## and ##\vec v = (b,d)## are linearly dependent
 
Cheers!
 
Question: A clock's minute hand has length 4 and its hour hand has length 3. What is the distance between the tips at the moment when it is increasing most rapidly?(Putnam Exam Question) Answer: Making assumption that both the hands moves at constant angular velocities, the answer is ## \sqrt{7} .## But don't you think this assumption is somewhat doubtful and wrong?

Similar threads

Replies
2
Views
2K
  • · Replies 1 ·
Replies
1
Views
1K
Replies
5
Views
2K
  • · Replies 2 ·
Replies
2
Views
1K
Replies
2
Views
2K
  • · Replies 9 ·
Replies
9
Views
3K
  • · Replies 4 ·
Replies
4
Views
2K
  • · Replies 1 ·
Replies
1
Views
2K
  • · Replies 26 ·
Replies
26
Views
8K
  • · Replies 7 ·
Replies
7
Views
3K